LSAT and Law School Admissions Forum

Get expert LSAT preparation and law school admissions advice from PowerScore Test Preparation.

 Administrator
PowerScore Staff
  • PowerScore Staff
  • Posts: 8917
  • Joined: Feb 02, 2011
|
#40613
Complete Question Explanation
(The complete setup for this game can be found here: lsat/viewtopic.php?t=13252)

The correct answer choice is (D)

The question stem establishes that P :longline: H, which can be added to the sequence created in the first rule (along with the block from the second rule) to produce the following chain:
PT69_Game_#1_#5_diagram 1.png
This creates a very interesting scenario, because H can have been written no earlier than fourth, and S can be written no earlier than fifth. But, from question #4, H cannot have been written fourth, and from the last rule H cannot have been written fifth. Thus, H must have been written sixth and S must have been written seventh. This is sufficient information to prove that answer choice (D) cannot be true, and is thus correct.

As a side note, L must have been written fifth in this question, and the overall diagram for this question appears as:
PT69_Game_#1_#5_diagram 2.png
Why is it the case that L was written later than F, GP, and M? Because if L were written earlier than F, for example, then F, H, and S would all be written after L, causing L to violate the third rule.
You do not have the required permissions to view the files attached to this post.

Get the most out of your LSAT Prep Plus subscription.

Analyze and track your performance with our Testing and Analytics Package.